Matemática, perguntado por guinomgamer, 5 meses atrás

Ajuda com calculo de limites

Calcule o limite a seguir:

Anexos:

Soluções para a tarefa

Respondido por Vicktoras
5

Temos o seguinte limite:

 \:  \:  \:  \:  \:  \:  \:  \:  \:  \:  \:  \:  \:   \:  \: \lim_{x\to \infty} \left(\frac{x-1}{x+1} \right)^x \\

Primeiro, vamos fazer uma modificação nessa expressão, para isso vamos usar uma propriedade que envolve a constante (e). Essa propriedade, nos diz que:

 \:  \:  \:  \:  \:  \:  \:   \:  \:  \:  \:  \: \bullet \:  \:  f(x) = e {}^{ \ln(f(x))}

Usando isso no nosso limite, temos que:

\lim_{x\to \infty} e^{ \ln \left( \left(\frac{x-1}{x+1} \right) \right)^x}  \:  \to \:\lim_{x\to \infty} \: e {}^{x \:  . \:  \ln \left(\frac{x-1}{x+1} \right)} \\

Para "descer" o x, usei a propriedade dos expoentes nos logarítmos, dada por:

 \:  \:  \:  \:  \:  \:  \:  \:  \:  \:  \:  \bullet \:  \ln( {a}^{b} ) = b \:. \ln(a)

Tendo feito isso, vamos aplicar uma propriedade de limites, dada por:

 \:  \:  \:  \:  \:  \:  \:  \bullet \:  \lim_{x\to \infty} e {}^{f(x)}    =  e {}^{ \lim_{x\to \infty} f(x)}   \\

Aplicando no nosso limite:

 \:  \:  \:  \:  \:  \:  \:  \:  \:  \:  \:  \:  \:  \:  \:  \:  \:  \:  \:  e ^{\lim_{x\to\infty} x \: . \ln \left(\frac{x-1}{x+1} \right)}\\

Agora para resolver o limite vamos fazer uma manipulação algébrica que deixe essa expressão com uma indeterminação do tipo ∞/∞ ou 0/0, para que possamos usar a regra de L'Hôpital:

 e ^{\lim_{x\to\infty} x \: . \ln \left(\frac{x-1}{x+1} \right)} \:  \to \:  \: e {}^{\lim_{x\to\infty}\frac{ \ln \left(\frac{x-1}{x+1} \right) }{ \frac{1}{x} }} \\

Se substituirmos o valor a qual o "x" tende, certamente vamos obter uma indeterminação do tipo ∞/∞, então vamos aplicar a regra de L'Hôpital, ou seja, derivar o numerador e denominador:

 \frac{d}{dx}  \left( \frac{1}{x}  \right) =  -  \frac{1}{x {}^{2} }  \:  \: e \:  \:  \frac{d}{dx}  \ln\left(\frac{x-1}{x+1} \right) =   \frac{2}{( x - 1).(x + 1)} \\

Substituindo essas derivações no devido local:

e {}^{ \lim_{x\to \infty} \frac{ \frac{2}{(x + 1).(x - 1)} }{ -  \frac{1}{x {}^{2} } } } \:  \to \: e {}^{ \lim_{x\to\infty} \frac{2}{(x + 1).(x - 1)} .( - x {}^{2} )}  \\  \\ e {}^{\lim_{x\to\infty}  \frac{ - 2x {}^{2} }{x {}^{2} - 1 } } \:

Agora vamos dividir todos os termos por x², pois quando fizermos a substituição do valor a qual o "x" tende, poderemos utilizar o Teorema da divisão de um termo finito por outro infinito:

e {}^{\lim_{x\to\infty} \frac{ \frac{ - 2x {}^{2} }{x {}^{2} } }{ \frac{x {}^{2} }{x {}^{2} } -  \frac{1 }{x {}^{2} }  } }  \:  \to \: e {}^{\lim_{x\to\infty}  \frac{ - 2}{1 - \frac{1}{x {}^{2} }  }   }

Agora vamos substituir o valor a qual o "x" tende:

e {}^{\lim_{x\to\infty} \frac{ - 2}{1 -  \frac{1}{ \infty } } }  \:  \to \: e {}^{\lim_{x\to\infty}  \frac{ - 2}{1 - 0} }  \:  \to \: e {}^{\lim_{x\to\infty} - 2}  \\

O limite de uma constante é a própria constante, então:

e {}^{\lim_{x\to\infty} - 2}  \:  \to \:   \boxed{ \boxed{\boxed{e { }^{ - 2}  \: ou \:  \frac{1}{e {}^{2} } }}}

Portanto, podemos concluir que:

 \:  \:  \:  \:  \:  \:  \:  \: \boxed{ \boxed{\lim_{x\to\infty}\left(\frac{x-1}{x+1} \right) {}^{x}  =  \frac{1}{e {}^{2} } }}

Espero ter ajudado

Respondido por CyberKirito
1

\large\boxed{\begin{array}{l}\displaystyle\sf\lim_{x \to \infty}\bigg(\dfrac{x-1}{x+1}\bigg)^x\\\\\displaystyle\sf\lim_{x \to \infty}\bigg(\dfrac{\backslash\!\!\!x\cdot\bigg[1-\dfrac{1}{x}\bigg]}{\backslash\!\!\!x\cdot\bigg[1+\dfrac{1}{x}\bigg]}\bigg)^x\\\\\sf\bigg[\dfrac{\displaystyle\lim_{x \to \infty}\bigg(1-\frac{1}{x}\bigg)}{\displaystyle\lim_{x \to \infty}\bigg(1+\frac{1}{x}\bigg)}\bigg]^x\end{array}}

\large\boxed{\begin{array}{l}\sf\dfrac{\displaystyle\lim_{x \to \infty}\bigg(1-\frac{1}{x}\bigg)^x}{\displaystyle\lim_{x \to \infty}\bigg(1+\dfrac{1}{x}\bigg)^x}\end{array}}

\large\boxed{\begin{array}{l}\displaystyle\sf\lim_{x \to\infty}\bigg(1-\dfrac{1}{x}\bigg)^x\\\underline{\rm fac_{\!\!,}a}\\\sf \dfrac{1}{k}=-\dfrac{1}{x}\\\sf x=-k\\\sf x\to\infty\,se\,k\to-\infty \\\displaystyle\sf\lim_{x \to \infty}\bigg(1-\dfrac{1}{x}\bigg)^x=\lim_{k\to-\infty}\bigg(1+\dfrac{1}{k}\bigg)^{-k}=\bigg[\lim_{x \to -\infty}\bigg(1+\dfrac{1}{k}\bigg)^k\bigg]^{-1}\\\sf =e^{-1}\end{array}}

\large\boxed{\begin{array}{l}\sf\dfrac{\displaystyle\lim_{x \to \infty}\bigg(1-\dfrac{1}{x}\bigg)^x}{\displaystyle\lim_{x \to \infty}\bigg(1+\dfrac{1}{x}\bigg)^x}=\dfrac{e^{-1}}{e}=e^{-1-1}=e^{-2}=\dfrac{1}{e^2}\end{array}}

\huge\boxed{\boxed{\boxed{\boxed{\displaystyle\sf\lim_{x \to \infty}\bigg(\dfrac{x-1}{x+1}\bigg)^x=\dfrac{1}{e^2}}}}}

Perguntas interessantes